Aren't quadratics larger?

Find the sum of all (possibly negative) integers n n such that

n 2 + 2 2014 n + 2. n^2+2 \mid 2014n+2.


The answer is 2013.

This section requires Javascript.
You are seeing this because something didn't load right. We suggest you, (a) try refreshing the page, (b) enabling javascript if it is disabled on your browser and, finally, (c) loading the non-javascript version of this page . We're sorry about the hassle.

3 solutions

Melanka Saroad
Aug 17, 2014

n^2+2 | (2014n+2)(2014n-2) then we get n^2+2 | 2*2014^2+4 but we know that 4 does not divide n^2+2 therefore n^2+2 | 2014^2+2. Now find the factors of 2014^2+2. Observe that the solutions of n obtained are such that n^2+2 | 2014n+2 or n^2+2 | 2014n-2 . So check validity of solutions. 0, 1, -2 and 2014 are the solutions.

Nice Solution and nice question @Finn Hulse :)

Happy Melodies - 6 years, 9 months ago
Aditya Raut
Aug 14, 2014

Once you start learning programming (and that too by self study), it does addict you, this is the way I used to solve the problem

img img

Sum of all of them is 2013 \boxed{2013}

truly the unappreciated way, but that gave a practice of programming....

i want a mathematical solution not a CS program for this question

Rishabh Jain - 6 years, 10 months ago

Log in to reply

See @Melanka Saroad 's solution. This problem has a standard approach.

Calvin Lin Staff - 6 years, 9 months ago

Can you justify that n n must be within that range?

Restricting the range of n n is a good way to approach this question.

Calvin Lin Staff - 6 years, 10 months ago

Log in to reply

It's easy to see from the question that n 2 + 2 2014 n + 2 n^2+2 \leq \left| 2014n+2 \right| , thus we get that for positive or 0 0 value of n n , n 2 2014 n n^2 \leq 2014n , and so for negative, It will be n 2 + 2 2014 n 2 n^2+2\leq -2014n-2 , a quadratic n 2 + 2014 n + 4 0 n^2+2014n+4\leq 0

so the range is reduced. But as i had an idea of this range,to reduce those many calculations, I directly kept a big range, no matter, because the needed range is surely going to be in that :P

Aditya Raut - 6 years, 10 months ago

Log in to reply

It is not true for n=3, if n=2015 how can n 2 n^{2} +2 <=|2014n+2|?

shuvayan ghosh dastidar - 6 years, 9 months ago

Log in to reply

@Shuvayan Ghosh Dastidar Please! Bigger positives don't divide smaller numbers' absolute value ! WE ARE GIVEN n 2 + 2 2014 n + 2 n^2+2 \mid 2014n +2 and that does mean that n 2 + 2 2014 n + 2 n^2+2 \leq 2014n +2

Aditya Raut - 6 years, 9 months ago

Nice! I really love CS approaches, I'm an admirer of anybody who can program. :D

Finn Hulse - 6 years, 10 months ago

Log in to reply

Is there a non-programming method?

Daniel Liu - 6 years, 10 months ago

Log in to reply

Of course dude.

Finn Hulse - 6 years, 10 months ago

Log in to reply

@Finn Hulse I also added a note that n n may be negative because I usually don't see negative numbers with the \mid sign.

Daniel Liu - 6 years, 10 months ago

I'm too lazy to add those integers. I too solved using Python. My code and output follows:

 1
 2
 3
 4
 5
 6
 7
 8
 9
10
11
>>> li=[]
>>> for n in range(-1000000, 1000000):
    if ((2014*n + 2)%(n**2 + 2)==0):
        li.append(n)
        print sum(li)


-2
-2
-1
2013

So answer is 2013

Nafees Zakir - 6 years, 8 months ago
Carsten Meyer
Mar 31, 2021

Define p ( n ) : = n 2 + 2 p(n):=n^2+2 and q ( n ) : = 2014 n + 2 q(n):=2014n+2 . Use Euclid's Algorithm to get possible common divisors: 2 100 7 2 p ( n ) ( 1007 n 1 ) q ( n ) = 4 100 7 2 + 2 = 2 3 787 859 = : N ( ) \begin{aligned} 2\cdot 1007^2p(n) - (1007n-1)q(n) &= 4\cdot 1007^2+2=2\cdot 3\cdot 787 \cdot 859=:N &&&(*) \end{aligned} Let's apply " m o d p ( n ) \mod p(n) " on both sides of ( ) (*) . We know p ( n ) p(n) divides q ( n ) q(n) , so the left part vanishes: N = 2 100 7 2 p ( n ) ( 1007 n 1 ) 0 m o d p ( n ) p ( n ) divides N \begin{aligned} N &= 2\cdot 1007^2p(n)-(1007n-1)\equiv 0 \mod p(n) &\Rightarrow && p(n) \text{ divides }N \end{aligned} Notice p ( n ) 2 p(n)\geq 2 must be a positive factor of N N - checking all 16 16 of them we only find integer solutions for p ( n ) 2 3 6 N n 0 ± 1 ± 2 ± 2014 \begin{array}{r|rrrr} p(n) & 2 & 3 & 6 & N\\\hline n & 0 & \pm 1 & \pm 2 & \pm 2014 \end{array} Find those n n that also satisfy " q ( n ) 0 m o d p ( n ) q(n)\equiv \red{0}\mod p(n) ": n 0 1 1 2 2 3 3 6 6 2014 2014 q ( n ) m o d p ( n ) 0 0 1 4 0 5 10 2 2 0 4 \begin{array}{r|rrrrrrrrrrr} n & 0 & 1 & -1 & 2 & -2 & 3 & -3 & 6 & -6 & 2014 & -2014\\\hline q(n)\mod p(n) & \red{0} & \red{0} & 1 & 4 & \red{0} & 5 & 10 & 2 & 2 & \red{0} & 4 \end{array} We have n { 0 ; 1 ; 2 ; 2014 } n\in\{0;\:1;\:-2;\:2014\} , and the answer is 0 + 1 2 + 2014 = 2013 0+1-2+2014=\boxed{2013}

0 pending reports

×

Problem Loading...

Note Loading...

Set Loading...